Use \dd for derivative d in Y&F-12 28.12 and 30
[course.git] / latex / problems / Serway_and_Jewett_4_venkat / problem28.V3.tex
1 \begin{problem}
2 Ultra-violet light of wavelength $\lambda$ incident on an emitter
3 surface gives rise to photoelectrons with maximum kinetic energy
4 $2.00\U{eV}$ whereas, for a wavelength $3\lambda/4$, the maximum
5 kinetic energy of emitted photoelectrons from the same surface is
6 $3.47\U{eV}$. \Part{a} Determine the wavelength $\lambda$ (in nm) of
7 these ultraviolet light waves incident on the emitter
8 surface. \Part{b} What are the photoelectric work function $\phi$ (in
9 eV) and the threshold wavelength $\lambda_\text{threshold}$ (in nm) of the
10 photons for photoelectron emission from this emitter surface. \Part{c} If
11 the wavelength of incident violet light is $400\U{nm}$, find the maximum
12 kinetic energy (in eV) of emitted photoelectrons.
13 \end{problem}
14
15 \begin{solution}
16 \Part{a}
17 Balancing energy in both cases
18 \begin{align}
19   \phi &= \frac{hc}{\lambda} - K_\text{max} = \frac{4hc}{3\lambda} - K_\text{max}' \\
20   K_\text{max}' - K_\text{max} &= \frac{1}{\lambda}(\frac{4}{3}hc-hc) \\
21   \lambda &= \frac{hc}{3(K_\text{max}' - K_\text{max})}
22     = \ans{281\U{nm}} \\
23   \frac{3}{4}\lambda = \ans{211\U{nm}}
24 \end{align}
25
26 \Part{b}
27 \begin{align}
28   \phi &= \frac{hc}{\lambda} - K_\text{max}
29     = \ans{2.41\U{eV}} \\
30   \phi &= \frac{hc}{\lambda_\text{threshold}} \\
31   \lambda_\text{threshold} &= \frac{hc}{\phi}
32     = \ans{514\U{nm}} \\ 
33 \end{align}
34
35 \Part{c}
36 \begin{equation}
37   K_\text{max} = \frac{hc}{\lambda} - \phi
38     = \ans{0.690\U{eV}}
39 \end{equation}
40 \end{solution}